diff --git "a/\350\256\262\344\271\211/\344\270\223\351\242\230/2 \347\272\277\346\200\247\347\251\272\351\227\264.tex" "b/\350\256\262\344\271\211/\344\270\223\351\242\230/2 \347\272\277\346\200\247\347\251\272\351\227\264.tex" index 05e3017..57c1449 100644 --- "a/\350\256\262\344\271\211/\344\270\223\351\242\230/2 \347\272\277\346\200\247\347\251\272\351\227\264.tex" +++ "b/\350\256\262\344\271\211/\344\270\223\351\242\230/2 \347\272\277\346\200\247\347\251\272\351\227\264.tex" @@ -239,7 +239,7 @@ \section{线性空间的定义} & = \lambda(a_1+b_1\i)+\lambda(a_2+b_2\i) = \lambda \alpha_1 + \lambda \alpha_2. \end{align*} \end{enumerate} - 所以$\mathbf{C}(\mathbf{C})$和$\mathbf{C}(\mathbf{R})$均构成线性空间.同理我们应当对八条性质逐条验证,但我们在第一讲以及说明了全体复数构成一个域,因此$\mathbf{C}(\mathbf{C})$自动满足线性空间的所有条件,此处不再赘述. 除此之外,$\mathbf{C}(\mathbf{R})$的加法运算与实数无关(回顾线性空间定义,实数只用来参与数乘运算),因此加法Abel群事实上与$\mathbf{C}(\mathbf{C})$一致,都是群$\langle \mathbf{C}:+\rangle$,此处也不再验证. 因此这里只验证$\mathbf{C}(\mathbf{R})$数乘运算是否满足线性空间定义的要求: + 所以$\mathbf{C}(\mathbf{C})$和$\mathbf{C}(\mathbf{R})$均构成线性空间. 同理我们应当对八条性质逐条验证,但我们在第一讲以及说明了全体复数构成一个域,因此$\mathbf{C}(\mathbf{C})$自动满足线性空间的所有条件,此处不再赘述. 除此之外,$\mathbf{C}(\mathbf{R})$的加法运算与实数无关(回顾线性空间定义,实数只用来参与数乘运算),因此加法Abel群事实上与$\mathbf{C}(\mathbf{C})$一致,都是群$\langle \mathbf{C}:+\rangle$,此处也不再验证. 因此这里只验证$\mathbf{C}(\mathbf{R})$数乘运算是否满足线性空间定义的要求: \begin{enumerate} \item 取定 $1 \in \mathbf{R}, \forall \alpha = a+b\i \in \mathbf{C},\enspace a, b \in \mathbf{R},\enspace 1 \cdot \alpha = 1 \cdot (a+b\i) = a+b\i = \alpha$. @@ -432,9 +432,9 @@ \section{线性子空间} \begin{proof} \begin{enumerate} \item 必要性显然,否则 $W$ 关于 $V$ 的运算不构成线性空间,所以只需证明充分性. - \item 由于 $W$ 是 $V$ 的子集,所以 $V(\mathbf{F})$ 中数乘满足的4条性质及加法的交换律与结合律对 $W$ 都成立.因此只要再证 $V$ 的零元 $0\in W$,$W$中每个元 $\alpha$ 的负元 $(-\alpha)\in W$.由于 $W$ 对数乘封闭,所以取 $\lambda=0$和$\lambda=-1$,即得: - \[ 0\cdot\alpha=0\in W,(-1)\cdot\alpha=-\alpha\in W \] - 故 $W$ 是 $V(\mathbf{F})$ 的线性子空间. + \item 由于 $W$ 是 $V$ 的子集,所以 $V(\mathbf{F})$ 中数乘满足的4条性质及加法的交换律与结合律对 $W$ 都成立. 因此只要再证 $V$ 的零元 $0\in W$,$W$中每个元 $\alpha$ 的负元 $(-\alpha)\in W$. 由于 $W$ 对数乘封闭,所以取 $\lambda=0$和$\lambda=-1$,即得: + \[ 0\cdot\alpha=0\in W,(-1)\cdot\alpha=-\alpha\in W \] + 故 $W$ 是 $V(\mathbf{F})$ 的线性子空间. \end{enumerate} \end{proof} @@ -599,31 +599,31 @@ \section{线性表示 \quad 线性扩张} (\lambda \circ f)(x) = \lambda f(x) \end{gather*} - \item \[V_1= \{f \mid x \in \mathbf{R}, f(x) \in \mathbf{R}, f(-x)=-f(x)\}\] + \item \[V_1= \{f \mid x \in \mathbf{R}, f(x) \in \mathbf{R}, f(-x)=-f(x)\}\] \[V_2= \{f \mid x \in \mathbf{R}, f(x) \in \mathbf{R}, f(0)=1, f(-x)=f(x)\}\] 对题(4)所定义的加法和数量乘法. - \item $V = \{f \mid x \in \mathbf{R}, f(x) \in \mathbf{C}, f(-x)= \overline{f(x)} \}$. - 对题(4)所定义的加法和数量乘法. + \item $V = \{f \mid x \in \mathbf{R}, f(x) \in \mathbf{C}, f(-x)= \overline{f(x)} \}$. + 对题(4)所定义的加法和数量乘法. \end{enumerate} \item 判断下列子集是否为给定线性空间的子空间: - \begin{enumerate} + \begin{enumerate} \item $W = \{(x_1,\ldots,x_n) \in F^n \mid a_1 x_1+\cdots +a_n x_n =0\}$, 其中 $a_1, \ldots, a_n$ 为域 $F$ 中的固定数量. - + \item $W_1 = \{(x,1,0) \in \mathbf{R}^3 \}$, $W_2 = \{(x,y,0) \in \mathbf{R}^3\}$. - + \item $W_1 = \{(x,y,z) \in \mathbf{R}^3 \mid x-3y+z = 0\}$, $W_2 = \{(x,y,z) \in \mathbf{R}^3 \mid x-3y+z = 1\}$. - - \item $W_1 = \left\{(x,y,z) \in \mathbf{R}^3 \middle|\ \dfrac{x}{2} = \dfrac{y-4}{1} = \dfrac{z-1}{-3}\right\}$, - - $W_2 = \{(x,y,z) \in \mathbf{R}^3 \mid x-y = 0, x+y+z = 0\}$. - + + \item $W_1 = \left\{(x,y,z) \in \mathbf{R}^3 \middle|\ \dfrac{x}{2} = \dfrac{y-4}{1} = \dfrac{z-1}{-3}\right\}$, + + $W_2 = \{(x,y,z) \in \mathbf{R}^3 \mid x-y = 0, x+y+z = 0\}$. + \item $W_1 = \{p(x) \in R[x] \mid p(1) = 0\}$, $W_2 = \{P(x) \in R[x]_n \mid p(1) = p(0)\}$(此题主要就是要判断满足一定条件的多项式是否构成子空间). - + \item $W = \{f \in F(-\infty, +\infty) \mid f(-x)=f(x), \forall x \in R\}$, 其中 $F(-\infty, +\infty)$ 是所有定义在 $(-\infty, +\infty)$ 上的实值函数对通常的函数加法及数与函数的乘法在实数域上构成的线性空间. - - \end{enumerate} + + \end{enumerate} \end{exgroup} \begin{exgroup} diff --git "a/\350\256\262\344\271\211/\344\270\223\351\242\230/7 \347\237\251\351\230\265\350\277\220\347\256\227\345\237\272\347\241\200.tex" "b/\350\256\262\344\271\211/\344\270\223\351\242\230/7 \347\237\251\351\230\265\350\277\220\347\256\227\345\237\272\347\241\200.tex" index 740a6e0..c04a720 100644 --- "a/\350\256\262\344\271\211/\344\270\223\351\242\230/7 \347\237\251\351\230\265\350\277\220\347\256\227\345\237\272\347\241\200.tex" +++ "b/\350\256\262\344\271\211/\344\270\223\351\242\230/7 \347\237\251\351\230\265\350\277\220\347\256\227\345\237\272\347\241\200.tex" @@ -12,29 +12,29 @@ \subsection{矩阵乘法的定义与基本性质} \[c_{ij}=a_{i1}b_{1j}+a_{i2}b_{2j}+\cdots+a_{im}b_{mj}\enspace(i=1,\ldots,p,\enspace j=1,\ldots,n).\] \end{definition} -这一定义带给我们的感受与我们在上一讲定义矩阵加法和数乘时的直观不同,在我们初看这一定义时必然会产生一个疑惑:为什么矩阵乘法定义得如此复杂,为什么不定义成两个矩阵对应元素相乘就可以了呢?事实上,这是因为矩阵乘法的定义来源于线性映射的复合的矩阵表示: +这一定义带给我们的感受与我们在上一讲定义矩阵加法和数乘时的直观不同,在我们初看这一定义时必然会产生一个疑惑:为什么矩阵乘法定义得如此复杂,为什么不定义成两个矩阵对应元素相乘就可以了呢?事实上,这是因为矩阵乘法的定义来源于线性映射的复合的矩阵表示: \begin{enumerate} \item 设线性空间 $V_1(F), V_2(F), V_3(F)$ 的基分别为 $B_1=\{\varepsilon, \varepsilon_2,\ldots,\varepsilon_n\},B_2=\{e_1,e_2,\ldots,e_m\},B_3=\{\zeta_1,\zeta_2,\ldots,\zeta_p\}$, $\sigma \in L(V_1,V_2), \tau \in L(V_2,v_3)$,且 $\sigma, \tau$ 分别关于基 $B_1$ 和 $B_2$ 及基 $B_2$ 和 $B_3$ 的矩阵为 $B=(b_{i,j})_{m \times n}$ 和 $A=(a_{ij})_{p\times m}$,即: - - $M(\tau)=A=\begin{pmatrix} - a_{11} & a_{12} & \cdots & a_{1m} \\ - a_{21} & a_{22} & \cdots & a_{2m} \\ - \vdots & & & \vdots \\ - a_{p1} & a_{p2} & \cdots & a_{pm} - \end{pmatrix}$, - $M(\sigma)=B=\begin{pmatrix} - b_{11} & b_{12} & \cdots & b_{1n} \\ - b_{21} & b_{22} & \cdots & b_{2n} \\ - \vdots & & & \vdots \\ - b_{m1} & b_{m2} & \cdots & b_{mn} - \end{pmatrix}$. + + $M(\tau)=A=\begin{pmatrix} + a_{11} & a_{12} & \cdots & a_{1m} \\ + a_{21} & a_{22} & \cdots & a_{2m} \\ + \vdots & & & \vdots \\ + a_{p1} & a_{p2} & \cdots & a_{pm} + \end{pmatrix}$, + $M(\sigma)=B=\begin{pmatrix} + b_{11} & b_{12} & \cdots & b_{1n} \\ + b_{21} & b_{22} & \cdots & b_{2n} \\ + \vdots & & & \vdots \\ + b_{m1} & b_{m2} & \cdots & b_{mn} + \end{pmatrix}$. \item 则 $\tau\sigma \in L(V_1,V_3)$ 关于基 $B_1$ 和 $B_3$ 的矩阵 $C=(c_{ij})_{p\times n}$ 中第 $j$ 列元素 $c_{1j},c_{2j},\ldots,c_{pj}$ 是 $\tau\sigma(\varepsilon_j)$ 在基 $B_3$ 下的坐标. 于是有: - - $\tau\sigma(\varepsilon_j)=\tau(\sigma(\varepsilon_j))=\tau(\sum_{k=1}^{m}b_{kj}e_k)=\sum_{k=1}^{m}b_{kj}\tau(e_k)=\sum_{k=1}^{m}b_{kj}(\sum_{i=1}^{p}a_{ik}\zeta_i)=\sum_{i=1}^{p}(\sum_{k=1}^{m}a_{ik}b_{kj})\zeta_i$. - 即得:$c_{ij}=a_{i1}b_{1j}+a_{i2}b_{2j}+\cdots+a_{im}b_{mj}$. + $\tau\sigma(\varepsilon_j)=\tau(\sigma(\varepsilon_j))=\tau(\sum_{k=1}^{m}b_{kj}e_k)=\sum_{k=1}^{m}b_{kj}\tau(e_k)=\sum_{k=1}^{m}b_{kj}(\sum_{i=1}^{p}a_{ik}\zeta_i)=\sum_{i=1}^{p}(\sum_{k=1}^{m}a_{ik}b_{kj})\zeta_i$. + + 即得:$c_{ij}=a_{i1}b_{1j}+a_{i2}b_{2j}+\cdots+a_{im}b_{mj}$. \end{enumerate} 接下来我们再给出两个线性映射的角度来理解矩阵乘法: @@ -79,21 +79,21 @@ \subsection{矩阵乘法的定义与基本性质} \begin{solution} $AB =\begin{pmatrix} - 0-3 & 3-1 \\ - 1-9 & 3+2-3 - \end{pmatrix}=\begin{pmatrix} - -3 & 2 & -8 & 2 - \end{pmatrix}$, + 0-3 & 3-1 \\ + 1-9 & 3+2-3 + \end{pmatrix}=\begin{pmatrix} + -3 & 2 & -8 & 2 + \end{pmatrix}$, $BA=\begin{pmatrix} - 0+3 & 0+3 & 0-9 \\ - 1+2 & 0+2 & -1-6 \\ - 3+1 & 0+1 & -3-3 - \end{pmatrix}=\begin{pmatrix} - 3 & 3 & -9 \\ - 3 & 2 & -7 \\ - 4 & 1 & -6 - \end{pmatrix}$. + 0+3 & 0+3 & 0-9 \\ + 1+2 & 0+2 & -1-6 \\ + 3+1 & 0+1 & -3-3 + \end{pmatrix}=\begin{pmatrix} + 3 & 3 & -9 \\ + 3 & 2 & -7 \\ + 4 & 1 & -6 + \end{pmatrix}$. \end{solution} 接下来给出矩阵运算的几个基本性质: @@ -106,7 +106,7 @@ \subsection{矩阵乘法的定义与基本性质} \item $(B+C)P=BP+CP$(右分配律) \end{enumerate} -证明方法十分简单暴力:直接设出矩阵元素然后暴力计算证明等号两边对应位置(如第$i$行第$j$列元素)相等即可. +证明方法十分简单暴力:直接设出矩阵元素然后暴力计算证明等号两边对应位置(如第$i$行第$j$列元素)相等即可. 实际上,由矩阵加法和乘法满足的运算律可知,全体$n$阶方阵构成的集合$\mathbf{F}^{n\times n}$关于矩阵加法和乘法构成环. @@ -380,7 +380,7 @@ \subsection{可逆的基本概念} \begin{proof} 只需证:若 $AB=E$,则必有 $BA=E$. 设 $A,B$ 分别是 $\sigma, \tau \in L(V,V)$ 关于 $V$ 的基 $B=\{\epsilon_1,\epsilon_2,\ldots,\epsilon_n\}$ 所对应的矩阵,则:$AB=E \iff \sigma\tau = I_V$. - + 因此要证明 $BA=E$,只需证明 $\tau\sigma=I_V$,由 $r(\sigma\tau)=r(I_V)=n \leq \min(r(\sigma),r(\tau))$,得:$r(\sigma)=r(\tau)=n$. 因此 $\forall \alpha \in V$,均存在唯一的 $\beta \in V$,使得 $\tau(\beta)=\alpha$,从而 $(\tau\sigma)(\alpha)=(\tau\sigma)(\tau(\beta))=\tau(\sigma\tau(\beta))=\tau(\beta)=\alpha$. @@ -424,31 +424,31 @@ \subsection{逆矩阵的求解(基本方法I)} \begin{solution} 以 $A$ 为系数矩阵的非齐次线性方程组 $AX=b$,对于任意的 $b=(b_1,b_2,b_3)$,可以用高斯消元法将其增广矩阵 - $$(A,b)=\left(\begin{array}{ccc:c} - 1 & -1 & 1 & b_1\\ - 0 & 1 & 2 & b_2\\ - 1 & 0 & 4 & b_3 - \end{array}\right)$$ 化为: + \[(A,b)=\left(\begin{array}{ccc:c} + 1 & -1 & 1 & b_1 \\ + 0 & 1 & 2 & b_2 \\ + 1 & 0 & 4 & b_3 + \end{array}\right)\] 化为: - $$\left(\begin{array}{ccc:c} - 1 & 0 & 0 & 4b_1+4b_2-3b_3\\ - 0 & 1 & 0 & 2b_1+3b_2-2b_3\\ - 0 & 0 & 1 & -b_1-b_2+b_3 - \end{array}\right)$$. + \[\left(\begin{array}{ccc:c} + 1 & 0 & 0 & 4b_1+4b_2-3b_3 \\ + 0 & 1 & 0 & 2b_1+3b_2-2b_3 \\ + 0 & 0 & 1 & -b_1-b_2+b_3 + \end{array}\right)\]. 因此,对任意的 $b$,方程组 $AX=b$ 有唯一解: - $$X=\begin{pmatrix}x_1\\x_2\\x_3\end{pmatrix}=\begin{pmatrix} - 4b_1+4b_2-3b_3\\2b_1+3b_2-2b_3\\-b_1-b_2+b_3 - \end{pmatrix}=\begin{pmatrix} - 4 & 4 & -3\\ - 2 & 3 & -2\\ - -1 & -1 & 1 - \end{pmatrix}\begin{pmatrix}b_1\\b_2\\b_3\end{pmatrix}$$ 即: - $$A^{-1}=\begin{pmatrix} - 4 & 4 & -3\\ - 2 & 3 & -2\\ - -1 & -1 & 1 - \end{pmatrix}$$ + \[X=\begin{pmatrix}x_1\\x_2\\x_3\end{pmatrix}=\begin{pmatrix} + 4b_1+4b_2-3b_3 \\2b_1+3b_2-2b_3\\-b_1-b_2+b_3 + \end{pmatrix}=\begin{pmatrix} + 4 & 4 & -3 \\ + 2 & 3 & -2 \\ + -1 & -1 & 1 + \end{pmatrix}\begin{pmatrix}b_1\\b_2\\b_3\end{pmatrix}\] 即: + \[A^{-1}=\begin{pmatrix} + 4 & 4 & -3 \\ + 2 & 3 & -2 \\ + -1 & -1 & 1 + \end{pmatrix}\] \end{solution} 关于逆矩阵的求解问题,我们将在介绍完初等变换后介绍第二种基本方法,剩余的进阶解法将在\nameref{chap:矩阵运算进阶}中介绍更多手段,以及我们会介绍矩阵方程求解的方法. 本节我们囿于一些计算技巧和基本概念暂未引入所以无法完全展开这些技巧. diff --git "a/\350\256\262\344\271\211/\344\270\223\351\242\230/8 \347\233\270\346\212\265\346\240\207\345\207\206\345\275\242.tex" "b/\350\256\262\344\271\211/\344\270\223\351\242\230/8 \347\233\270\346\212\265\346\240\207\345\207\206\345\275\242.tex" index f665450..799f5f1 100644 --- "a/\350\256\262\344\271\211/\344\270\223\351\242\230/8 \347\233\270\346\212\265\346\240\207\345\207\206\345\275\242.tex" +++ "b/\350\256\262\344\271\211/\344\270\223\351\242\230/8 \347\233\270\346\212\265\346\240\207\345\207\206\345\275\242.tex" @@ -430,40 +430,40 @@ \subsection{基本概念与性质} \end{enumerate} \end{definition} 三种初等矩阵具体形状如下: -% \[ -% E_i(c) = -% \kbordermatrix{ -% & 1 & & & & \\ -% & & \ddots & & & \\ -% \text{$i$ 行} & & & c & & \\ -% & & & & \ddots & \\ -% & & & & & 1 -% } -% \] - -% \[ -% E_{ij}(c) = -% \kbordermatrix{ -% & 1 & & & & & \\ -% & & \ddots & & & & \\ -% \text{$i$ 行} & & & 1 & \cdots & 1 & \\ -% & & c & 1 & & & \\ -% \text{$j$ 行} & & & & & \ddots & \\ -% & & & & & & 1 -% } -% \] - -% \[ -% E_{ij} = -% \kbordermatrix{ -% & 1 & & & & & \\ -% & & \ddots & & & & \\ -% \text{$i$ 行} & & & 0 & \cdots & 1 & \\ -% & & 1 & 0 & & & \\ -% \text{$j$ 行} & & & & & \ddots & \\ -% & & & & & & 1 -% } -% \] + +\begin{gather*} + E_i(c) = \begin{blockarray}{cccccc} + \begin{block}{[ccccc]c} + 1 & & & & & \\ + & \ddots & & & & \\ + & & c & & & {\footnotesize\text{第$i$行}} \\ + & & & \ddots & & \\ + & & & & 1 & \\ + \end{block} + \end{blockarray} \\ + E_{ij}(c) = \begin{blockarray}{cccccccc} + \begin{block}{[ccccccc]c} + 1 & & & & & & & \\ + & \ddots & & & & & & \\ + & & 1 & & & & & {\footnotesize\text{第$i$行}} \\ + & & & \ddots & & & & \\ + & & c & & 1 & & & {\footnotesize\text{第$j$行}} \\ + & & & & & \ddots & & \\ + & & & & & & 1 & \\ + \end{block} + \end{blockarray} \\ + E_{ij} = \begin{blockarray}{cccccccc} + \begin{block}{[ccccccc]c} + 1 & & & & & & & \\ + & \ddots & & & & & & \\ + & & 0 & & 1 & & & {\footnotesize\text{第$i$行}} \\ + & & & \ddots & & & & \\ + & & 1 & & 0 & & & {\footnotesize\text{第$j$行}} \\ + & & & & & \ddots & & \\ + & & & & & & 1 & \\ + \end{block} + \end{blockarray} +\end{gather*} 事实上,初等矩阵的定义就将我们在高斯消元法中使用的初等变换的三种形式对应到了矩阵的形式上. 我们很容易通过计算验证,简而言之,对单位矩阵$E$做了一次初等变换后得到的矩阵$P$,乘以其他任何矩阵$A$的效果就是对$A$做了和对$E$做的同样的初等变换. @@ -478,7 +478,7 @@ \subsection{基本概念与性质} \item 注意三类矩阵不是三个矩阵,例如倍乘矩阵乘以哪一行/哪一列,以及乘以多少都是不唯一的; - \item 三种初等矩阵都是可逆的,且$E_i^{-1}(c)=E_i\left(\dfrac{1}{c}\right)$,$E_{ij}^{-1}(c)=E_{ij}(-c)$,$E_{ij}^{-1}=E_{ij}$. 原因非常简单,只需要记住这三类矩阵在单位矩阵基础上做了什么,需要反过来作用什么来抵消就可以理解; + \item 三种初等矩阵都是可逆的,且$E_i^{-1}(c)=E_i(1/c)$,$E_{ij}^{-1}(c)=E_{ij}(-c)$,$E_{ij}^{-1}=E_{ij}$. 原因非常简单,只需要记住这三类矩阵在单位矩阵基础上做了什么,需要反过来作用什么来抵消就可以理解; \item 三种初等矩阵的转置:$E_i^\mathrm{T}(c)=E_i(c)$,$E_{ij}^\mathrm{T}(c)=E_{ji}(c)$,$E_{ij}^\mathrm{T}=E_{ij}$,因此初等矩阵转置前后分别对应于同样的行列变换操作. 例如倍乘行变换表示对第$i$行乘以$c$,转置后如果视为列变换则表示对第$i$列乘以$c$,行列操作一致,对换也是如此. 而倍加$E_{ij}(c)$在行变换表示将第$i$行乘$c$加到第$j$行,转置后如果视为列变换,$E_{ji}(c)$表示将第$i$列乘$c$加到第$j$列,这两者在行列操作上保持了一致性. @@ -529,63 +529,47 @@ \subsection{逆矩阵的求解(基本方法II)} \end{example} \begin{solution} - \[ -(A, E) = -\left( -\begin{array}{cccc:ccc} -0 & 2 & -1 & 1 & 0 & 0 & 0 \\ -1 & 1 & 2 & 0 & 1 & 0 & 0 \\ --1 & -1 & -1 & 0 & 0 & 0 & 1 -\end{array} -\right) -\xrightarrow{[1] \leftrightarrow [2]} -\left( -\begin{array}{cccc:ccc} -1 & 1 & 2 & 0 & 1 & 0 & 0 \\ -0 & 2 & -1 & 1 & 0 & 0 & 0 \\ --1 & -1 & -1 & 0 & 0 & 0 & 1 -\end{array} -\right) -\] - -\[ -\xrightarrow{[3] + [1]} -\left( -\begin{array}{cccc:ccc} -1 & 1 & 2 & 0 & 1 & 0 & 0 \\ -0 & 2 & -1 & 1 & 0 & 0 & 0 \\ -0 & 0 & 1 & 0 & 1 & 1 & 1 -\end{array} -\right) -\xrightarrow{[1] + [3] \times (-2) ; [2] + [3]} -\left( -\begin{array}{cccc:ccc} -1 & 1 & 0 & 0 & -1 & -1 & -2 \\ -0 & 2 & 0 & 1 & 1 & 1 & 1 \\ -0 & 0 & 1 & 0 & 1 & 1 & 1 -\end{array} -\right) -\] - -\[ -\xrightarrow{[2] \times \left(\dfrac{1}{2}\right) ; [1] + [2] \times (-1)} -\left( -\begin{array}{ccc:ccc} -1 & 0 & 0 & -\dfrac{1}{2} & -\dfrac{3}{2} & -\dfrac{5}{2} \\ -0 & 1 & 0 & \dfrac{1}{2} & \dfrac{1}{2} & \dfrac{1}{2} \\ -0 & 0 & 1 & 0 & 1 & 1 -\end{array} -\right) -\] - -故 $A^{-1} = -\left( -\begin{array}{ccc} --\dfrac{1}{2} & -\dfrac{3}{2} & -\dfrac{5}{2} \\ -\dfrac{1}{2} & \dfrac{1}{2} & \dfrac{1}{2} \\ -0 & 1 & 1 -\end{array} -\right)$. + \begin{gather*} + (A, E) = + \left(\begin{array}{ccc:ccc} + 0 & 2 & -1 & 1 & 0 & 0 \\ + 1 & 1 & 2 & 0 & 1 & 0 \\ + -1 & -1 & -1 & 0 & 0 & 1 + \end{array}\right) + \xrightarrow{[1] \leftrightarrow [2]} + \left(\begin{array}{ccc:ccc} + 1 & 1 & 2 & 0 & 1 & 0 \\ + 0 & 2 & -1 & 1 & 0 & 0 \\ + -1 & -1 & -1 & 0 & 0 & 1 + \end{array}\right) \\ + \xrightarrow{[3] + [1]} + \left(\begin{array}{ccc:ccc} + 1 & 1 & 2 & 0 & 1 & 0 \\ + 0 & 2 & -1 & 1 & 0 & 0 \\ + 0 & 0 & 1 & 0 & 1 & 1 + \end{array}\right) + \xrightarrow{\substack{[1] + [3] \times (-2) \\ [2] + [3]}} + \left(\begin{array}{ccc:ccc} + 1 & 1 & 0 & 0 & -1 & -2 \\ + 0 & 2 & 0 & 1 & 1 & 1 \\ + 0 & 0 & 1 & 0 & 1 & 1 + \end{array}\right) \\ + \xrightarrow{\substack{[2] \times (1/2) \\ [1] + [2] \times (-1)}} + \left( + \begin{array}{ccc:ccc} + 1 & 0 & 0 & -\frac{1}{2} & -\frac{3}{2} & -\frac{5}{2} \\ + 0 & 1 & 0 & \frac{1}{2} & \frac{1}{2} & \frac{1}{2} \\ + 0 & 0 & 1 & 0 & 1 & 1 + \end{array} + \right) + \end{gather*} + + 故 $A^{-1} = + \begin{pmatrix} + -\frac{1}{2} & -\frac{3}{2} & -\frac{5}{2} \\ + \frac{1}{2} & \frac{1}{2} & \frac{1}{2} \\ + 0 & 1 & 1 + \end{pmatrix}$. \end{solution} \section{初等矩阵与相抵标准形} @@ -598,7 +582,7 @@ \subsection{从初等变换到相抵标准形} 初等变换不改变矩阵的秩(包括行变换和列变换). \end{theorem} -定理的证明很简单,只需对各个初等变换逐一通过计算验证即可,具体证明如下: +定理的证明很简单,只需对各个初等变换逐一通过计算验证即可,具体证明如下: \begin{proof} 因为初等列变换和初等行变换是等价的,所以我们仅就初等行变换的情形给以证明. @@ -607,9 +591,9 @@ \subsection{从初等变换到相抵标准形} (1) 将 $A$ 的第 $i, j$ 行对换得到 $B$,则 $B$ 与 $A$ 的行向量组相同,故 $A, B$ 的行秩相等. - (2) 将 $A$ 的第 $i$ 行乘非零常数 $c$ 得到 $B$,则 $B$ 的行向量组:$\beta_i = c\alpha_i, \beta_k = \alpha_k \ (k \neq i)$.因此 $B$ 与 $A$ 的行向量组可以互相线性表示.所以 $A$ 与 $B$ 的行秩相等. + (2) 将 $A$ 的第 $i$ 行乘非零常数 $c$ 得到 $B$,则 $B$ 的行向量组:$\beta_i = c\alpha_i, \beta_k = \alpha_k \enspace (k \neq i)$. 因此 $B$ 与 $A$ 的行向量组可以互相线性表示. 所以 $A$ 与 $B$ 的行秩相等. - (3) 将 $A$ 的第 $i$ 行乘常数 $c$ 加到第 $j$ 行得到 $B$,则 $B$ 的行向量组:$\beta_j = c\alpha_i + \alpha_j, \beta_k = \alpha_k \ (k \neq j)$.相应地也有 $\alpha_j = \beta_j - c\beta_i, \alpha_k = \beta_k \ (k \neq j)$.因此 $A$ 与 $B$ 的行向量组可以互相线性表示.所以 $A$ 与 $B$ 的行秩相等. + (3) 将 $A$ 的第 $i$ 行乘常数 $c$ 加到第 $j$ 行得到 $B$,则 $B$ 的行向量组:$\beta_j = c\alpha_i + \alpha_j,\allowbreak \beta_k = \alpha_k \enspace (k \neq j)$. 相应地也有 $\alpha_j = \beta_j - c\beta_i, \alpha_k = \beta_k \enspace (k \neq j)$. 因此 $A$ 与 $B$ 的行向量组可以互相线性表示. 所以 $A$ 与 $B$ 的行秩相等. \end{proof} 由这一定理我们同样可以证明\autoref*{thm:相抵标准形},因为我们可以通过对任何一个矩阵做一系列初等行变换$P_1,\ldots,P_s$得到(行)简化阶梯矩阵,再做一系列初等列变换$Q_1,\ldots,Q_t$,即可将矩阵化为$U_r$的形式. @@ -641,15 +625,15 @@ \subsection{从初等变换到相抵标准形} 我们在这里对初等变换做一个小小的总结. 事实上初等变换只有三个非常重要的性质,即初等变换可逆,可逆矩阵可以写为初等变换的乘积,以及初等变换不改变矩阵的秩,只需牢记这三点就能覆盖几乎全部的证明技巧. \item 事实上,相抵也被称为等价,相抵标准形也被称为等价标准形,原因就在于相抵是矩阵的一个等价关系;证明如下: - \begin{enumerate} - \item $E_m A E_n = A$, 即 $A \cong A$; + \begin{enumerate} + \item $E_m A E_n = A$, 即 $A \cong A$; - \item 若 $A \cong B$, 则存在可逆矩阵 $P$ 和 $Q$, 使得 $PAQ = B$, 于是, $P^{-1} B Q^{-1} = A$, 从而 $B \cong A$; + \item 若 $A \cong B$, 则存在可逆矩阵 $P$ 和 $Q$, 使得 $PAQ = B$, 于是, $P^{-1} B Q^{-1} = A$, 从而 $B \cong A$; - \item 若 $A \cong B, B \cong C$, 则存在可逆矩阵 $P, S, Q, T$, 使得 $PAQ = B, SBT = C$, 于是就有 $(SP)A(QT) = C$, 从而 $A \cong C$. - \end{enumerate} - - 这里要强调的是,这一等价关系将矩阵空间$\mathbf{F}^{m\times n}$中的全体元素按秩进行了分类,每一类对应的相抵标准形都是一样的. + \item 若 $A \cong B, B \cong C$, 则存在可逆矩阵 $P, S, Q, T$, 使得 $PAQ = B, SBT = C$, 于是就有 $(SP)A(QT) = C$, 从而 $A \cong C$. + \end{enumerate} + + 这里要强调的是,这一等价关系将矩阵空间$\mathbf{F}^{m\times n}$中的全体元素按秩进行了分类,每一类对应的相抵标准形都是一样的. \end{enumerate} \begin{example}{}{} diff --git "a/\350\256\262\344\271\211/\347\272\277\346\200\247\344\273\243\346\225\260\350\215\243\350\252\211\350\257\276\350\276\205\345\255\246\350\256\262\344\271\211.tex" "b/\350\256\262\344\271\211/\347\272\277\346\200\247\344\273\243\346\225\260\350\215\243\350\252\211\350\257\276\350\276\205\345\255\246\350\256\262\344\271\211.tex" index 6270b37..b190513 100644 --- "a/\350\256\262\344\271\211/\347\272\277\346\200\247\344\273\243\346\225\260\350\215\243\350\252\211\350\257\276\350\276\205\345\255\246\350\256\262\344\271\211.tex" +++ "b/\350\256\262\344\271\211/\347\272\277\346\200\247\344\273\243\346\225\260\350\215\243\350\252\211\350\257\276\350\276\205\345\255\246\350\256\262\344\271\211.tex" @@ -11,6 +11,7 @@ \usepackage{booktabs} % Excel 导出的大表格 \usepackage{rotating} \usepackage{extarrows} +\usepackage{blkarray} \usepackage{float} \usepackage{diagbox}